You are on page 1of 21

FIITJEE PATNA CENTRE

CLIP
CHAPTER

MAGNETISM PAJ

Problem 1. A particle of mass m having a charge q enters into a


circular region of radius R with velocity v directed towards the centre. The R
strength of magnetic field is B. Find the deviation in the path of the particle.

Problem 2. A ring of mass m and radius r carrying current I 0 is lying v

in the x-y plane with centre at the origin. A uniform magnetic field of strength y
B 0 (2i$ − 3$j + 5 k$) T is applied in the region. If the ring can rotate about the axis I0 A'
AA′ in x-y plane only, find :
(a) initial angular acceleration.
R 45°
(b) the initial magnetic energy stored in the ring.
O x
(c) the force on the loop.

A
Problem 3. A particle of specific charge α (charge per unit mass) is

released at time t = 0 from origin with an initial velocity of v = v 0i$ in a uniform magnetic field

B = – B 0 k$. Find the velocity and position of particle at any time t.

Problem 4. A positively charged particle having charge q1 = 1 C and mass m1 = 40 g is revolving


along a circle of radius R = 40 cm with speed v1 = 5 m / s in a uniform magnetic field with centre of circle
at origin. At time t = 0, the particle is at (0, 0.4 m, 0 ) and velocity is directed along positive x-direction.
Another particle having charge q2 = 1 C and mass m 2 = 10 g moving uniformly parallel to z-direction
with velocity v 2 = 40 m / s collides with revolving particle at t = 0 and gets stuck to it. Neglecting
π
gravitational force and coulomb force calculate x, y and z co-ordinates of the combined particle at t =
40
second.
Problem 5. A particle with specific charge α (charge per unit mass) moves in the region of space
→ →
where mutually perpendicular electric field E = E 0 $j and magnetic field B = B 0k$ are present. At time
t = 0particle is located at origin with zero initial velocity. Find x and y co-ordinates of particle at any time t.
Problems 93
→ → →
Problem 487. A hypothetical magnetic field existing in a region is given by B = B 0 e r , where e r
denotes the unit vector along the radial direction. A circular loop of radius ' a' carrying a current i, is
placed with its plane parallel to the x-y plane and the centre at (0,0, d). Find the magnitude of the
magnetic force acting on the loop.

Problem 7. A regular polygon of n side is formed by bending a wire of total length 2 πr which
carries a current i.
(a) Find the magnetic field B at the centre of the polygon.
(b) By letting n → ∞, deduce the expression for the magnetic field at the centre of a circular coil.
Problem 8. Figure shows a circular wire loop of radius a carrying a

+
current i, placed in a perpendicular magnetic field B
(a) consider a small part dl of the wire, find the force on this part of the

+
wire exerted by the magnetic field.
(b) find the force of compression in the wire
a

+
(c) suppose that the radius of cross-section of the wire used is r. Find the
increase in the radius of the loop if the magnetic field is switched off.
The Young's modulus of the material of the wire is Y.

+
i

2
Problem 9. In the configuration shown in figure wires 1 and 2 are 1
+
i2
infinitely long and carry currents i1 and i 2 in a direction perpendicular to +
plane of paper and inwards. Wire 3 of length l carries a current i 3 from left to i1
l2
right. Find the net magnetic force on wire 3. l1

i3
3
l

Problem 491. A rod of mass 072 . kg and radius 6 cm rests on two


B
parallel rails, that are d = 12 cm apart and L = 45 cm long. The rod
d
carries a current i = 48 A (in the direction shown) and rolls along the
rails without slipping. If it starts from rest, what is the speed of the rod i
as it leaves the rails if a uniform magnetic field of magnitude 0 . 24 T is
directed perpendicular to the rod and the rails.
L

Problem 492. Magnetic field B = – B 0 x k$ exists in a region of space. A particle of specific charge α

(charge per unit mass) enters this region of space. Its velocity and position at time t = 0, are v = v 0 i$ and
(0, 0, 0). Find the maximum x-displacement of the particle.

Problem 493. Electric field and magnetic field in a region of space are given by and E = E 0 $j and

B = B 0 $j . A particle of specific charge α (charge per unit mass) is released from origin with velocity

v = v 0i$. Determine
(a) the path of the particle
(b) velocity of particle at any time t
(c) position of particle at time t
(d) the y coordinate of particle when it crosses the y-axis for the n th time and
(e) angle φ between particle’s velocity and y-axis at that moment
P1

Problem 13. A long cylindrical conductor of radius a has two


cylindrical cavities of diameter a through its entire length as shown in r
cross-section in figure. A current I is directed out of the page and is uniform a
throughout the cross-section of the conductor. Find the magnitude and
P2
direction of the magnetic field in terms of µ 0 , I, r and a.
(a) at point P1 and a
(b) at point P2
r

Problem 14. A proton and an alpha particle, after being accelerated through same potential
difference, enter a uniform magnetic field the direction of which is perpendicular to their velocities.
Find the ratio of radii of the circular paths of the two particles. (JEE 2004)
Problem 15. A ring of radius R having uniformly distributed
charge Q is mounted on a rod suspended by two identical strings. The D
tension in strings in equilibrium is T0 . Now a vertical magnetic field is
switched on and ring is rotated at constant angular velocity ω. Find R
A
the maximum ω with which the ring can be rotated if the strings can ω
3T
withstand a maximum tension of 0 ⋅
2 (JEE 2003) B

Problem 16. A rectangular loop PQRS made from a uniform wire z


has length a, width b and mass m. It is free to rotate about the arm PQ,
which remains hinged along a horizontal line taken as the y-axis (see
figure). Take the vertically upward direction as the z-axis. A uniform
→ P
magnetic field B = ( 3i$ + 4k$ ) B0 exists in the region. The loop is held in the Q y

x-y plane and a current I is passed through it. The loop is now released a
and is found to stay in the horizontal position in equilibrium. S
x R
b
(a) What is the direction of the current I in PQ?
(b) Find the magnetic force on the arm RS.
(c) Find the expression for I in terms of B0 , a, b and m.
Problem 17. A current of 10 A flows around a closed path in a
circuit which is in the horizontal plane as shown in the figure. The circuit D

consists of eight alternating arcs of radii r1 = 0.08 m and r2 = 0.12 m. C


r2
Each subtends the same angle at the centre.
(a) Find the magnetic field produced by this circuit at the centre.
A
(b) An infinitely long straight wire carrying a current of 10 A is r1
passing through the centre of the above circuit vertically with the
direction of the current being into the plane of the circuit. What
is the force acting on the wire at the centre due to the current in
the circuit ? What is the force acting on the arc AC and the
straight segment CD due to the current at the centre ? (JEE 2001)
Problem 18. A circular loop of radius R is bent along a diameter and given a shape as shown in
figure. One of the semicircles ( KNM ) lies in the X-Z plane and the other one ( KLM ) in the Y-Z plane
with their centres at origin. Current I is flowing through each of the semicircles as shown in figure.

L Y
I M

N X

I
K Z


(a) A particle of charge q is released at the origin with a velocity v = – v 0i$. Find the

instantaneous force F on the particle. Assume that space is gravity free.
→ →
(b) If an external uniform magnetic field B0 $j is applied, determine the force F1 and F2 on the

semicircles KLM and KNM due to the field and the net force F on the loop. (JEE 2000)

Problem 19. The region between x = 0 and x = L is filled with uniform steady magnetic field
B0 k$. A particle of mass m, positive charge q and velocity v 0i$ travels along x-axis and enters the region
of the magnetic field.
Neglect the gravity throughout the question
(a) Find the value of L if the particle emerges from the region of magnetic field with its final
velocity at an angle 30° to its initial velocity.
(b) Find the final velocity of the particle and the time spent by it in the magnetic field, if the
magnetic field now expands upto 2.1 L. (JEE 1999)

Problem 20. A particle of mass m and charge q is moving in a region where uniform, constant
→ → → →
electric and magnetic fields E and B are present. E and B are parallel to each other. At time t = 0,
→ →
the velocity v 0 of the particle is perpendicular to E (Assume that its speed is always << c, the speed of

light in vacuum). Find the velocity v of the particle at time t. You must express your answer in terms
→ → →
of t, q, m, the vectors v 0 , E and B and their magnitudes v 0 , E and B. (JEE 1998)

Problem 21. A uniform constant magnetic field B is directed at an angle of 45°to the X-axis in
X – Y plane. PQRS is a rigid square wire frame carrying a steady current I 0 , with its centre at the
origin O. At time t = 0, the frame is at rest in the position shown in the figure with its sides parallel to X
and Y axes. Each side of the frame is of mass M and length L.

I0
S R

P Q


(a) What is the torque τ about O acting on the frame due to the magnetic field ?
(b) Find the angle by which the frame rotates under the action of this torque in a short interval
of time ∆t, and the axis about which this rotation occurs (∆t is so short that any variation in
the torque during this interval may be neglected). Given : the moment of inertia of the frame
4
about an axis through its centre perpendicular to its plane is ML2 .
3
Problem 22. Three infinitely long thin wires, each carrying current i in the same direction, are
in the X-Y plane of a gravity free space. The central wire is along the y-axis while the other two are
along x = ± d.
(i) Find the locus of the points for which the magnetic field B is zero.
(ii) If the central wire is displaced along the z-direction by a small amount and released, show
that it will execute simple harmonic motion. If the linear density of the wires is λ, find the
frequency of oscillation. (JEE 1997)

Problem 23. An infinitesimally small bar magnet of dipole moment M pointing and moving
with the speed v in the X-direction. A small closed circular conducting loop of radius a and negligible
self-inductance lies in the Y-Z plane with its centre at x = 0, and its axis coinciding with the X-axis.
Find the force opposing the motion of the magnet, if the resistance of the loop is R. Assume that the
distance x of the magnet from the centre of the loop is much greater than a. (JEE 1997, Cancelled)
Problem 24. An electron in the ground state of hydrogen atom is B
n
revolving in anticlockwise direction in a circular orbit of radius R (figure).
(i) Obtain an expression for the orbital magnetic moment of the θ

electron.

(ii) The atom is placed in a uniform magnetic induction B such that the
plane normal of the electron orbit makes an angle of 30° with the
magnetic induction. Find the torque experienced by the orbiting
electron. (JEE 1996)

Problem 25. A long horizontal wire AB, which is free to move in a


A B
vertical plane and carries a steady current of 20 A, is in equilibrium at a
height of 0.01 m over another parallel long wire CD which is fixed in a
C D
horizontal plane and carries a steady current of 30 A, as shown in
figure. Show that when AB is slightly depressed, it executes simple
harmonic motion. Find the period of oscillations.
Problem 26. An electron gun G emits electrons of energy 2 keV
S
travelling in the positive X-direction. The electrons are required to hit the
B
spot S where GS = 0.1 m, and the line GS make an angle of 60° with the

x-axis as shown in figure. A uniform magnetic field B parallel to GS exists in G
60°
X

the region outside the electron gun. Find the minimum value of B needed to
make the electrons hit S.

Problem 27. A straight segment OC (of length L) of a circuit carrying a current i is placed along
the X-axis as shown in figure. Two infinitely long straight wires A and B, each extending from z = – ∞
to + ∞, are fixed at y = – a and y = + a respectively, as shown in the figure. If the wires A and B each
carry a current i into the plane of the paper, obtain the expression for the force acting on the segment
OC. What will be force on OC if the current in the wire B is reversed ? (JEE 1992)

i C
O X

Z
CHAPTER

Hints & Solutions


1. v is perpendicular to B 3. Magnetic field is in negative Z-direction i.e.,
mv 2 perpendicular to paper inwards. Velocity
∴ = qvB vector is perpendicular to magnetic field,
r
therefore, path of the particle will be a circle.
 mv  δ Time period of which is given by
r=  O
 qB  Y
θ
Where r is the radius of θ →
B
circular trajectory of
the particle. Where c is C
the centre of its circular X

path.
r
Deviation δ = (π − 2θ) = π − 2 tan −1   Z
 R
2π m 2π q 
T = =  = α
−1  mv  B0q B0α m 
= π − 2 tan  
 qBR  2π
or angular velocity ω = = B0α …(1)
2. (a) Magnetic moment of the ring T
→ Radius of the circle is
M = πr 2I 0 k$
mv 0 v
→ r= = 0 …(2)
B = B0 (2i$ − 3 $j + 5k$) B0q B0α
→ → In time t, the particle will rotate an angle
⇒ Torque = M × B = I 0B0 πr 2 (3i$ + 2 $j ) θ = ωt = B0αt as shown in figure.
(3i$ + 2 $j )• (i$ + $j ) Y
v0
⇒ τ AA′ = I 0B0 πr 2
2 →
B θ
τ AA′
∴ α AA′ = t=t
mr 2/ 2
0 X
5 2I 0 B 0 π v0
= rad /s2 t=0
m Hence velocity of particle at time t would be
→ →
(b) U = − M • B →
v = v 0 cos θ $i + v 0 sin θ $j
= − [(πr 2I 0) k$ • B0 (2i$ − 3 $j + 5k$)] →
= − πr 2I 0B0(5) or v = v 0 cos ( B0αt) $i + v 0 sin ( B0αt) $j

⇒ U = − 5π r 2 I 0 B 0 J Similarly position of particle can be found as :



(c) In uniform magnetic field force on a rp = x$i + y$j
current carrying loop is zero.
where x = r sin θ and y = r (1 – cos θ)
Problems in Physics
Y →
or v = 4 i$ + 8 k$
θ r v0
Since velocity of the combined mass makes
r B some angles (≠ 0° , 180° or 90° ) with the
P
magnetic field. Hence the path of the
y combined mass is helical. Radius of which is
(m + m2) v x
R′ = 1
v0
X (q1 + q2) B
x (0.05) (4)
= = 0.2 m
v (2) (0.5)
where r= 0
B0α Time period of revolution is
and θ = B0αt 2π (m1 + m2)
T =
→ v (q1 + q2) B
Hence rp = 0 [sin ( B0αt) $i
B0α 2π (0.05) π
= = second
+ {1 – cos ( B0αt)} $j ] (2) (0.5) 10
π T
4. At time t = 0particle 1 is at (0, 0.4m, 0) and its The given time t = s=
40 4
velocity is along positive x-direction and In this time the combined mass will rotate an
→ π
magnetic force F mis towards origin or along angle θ = or quarter circle in x − y plane.
2
negative y-direction as shown. Hence
But now the radius will become 0.2 m
according to Fleming’s left hand rule, the
→ So x = R = 0.2 m ⇒ y = R − R ′ = 0.2 m
magnetic field B should be along positive  π
z-direction. Also and z = v z t = (8)   = 0.628 m
Y  40
t=0 v1 π
∴ Position of combined mass at t = s is
Fm 40
(0.2 m, 0.2 m, 0.628 m)
X 5. . Since the magnetic field is in positive
O z-direction, magnetic force will be in x-y plane.
Electric field is in positive y-direction.
Therefore, electrostatic force will be in
y-direction i.e., component of force along
z-direction is always zero. Therefore, at any
m1v1
R= moment its velocity will have x and y
Bq1 components only.
m1v1 (0.04) (5) Let at time t velocity of particle be
∴ B= = = 0.5 tesla →
Rq1 (0.40 (1)
v = v x $i + v y $j
Now applying conservation of linear
momentum before and after collision, we get Therefore, the Lorentz force on the particle
→ → → → → → →
(m1 + m2) v = m1 v1 + m2 v 2 F = q { E + ( v × B )}
= q [E 0 $j + {(v x $i + v y $j) × (B0 k$ )}]

or (0.04 + 0.01) v = (0.04) 5 i$ + (0.01)(40) k$ = q [(E 0 – v x B0) $j + B0 v y $i ]
Solutions 363
→ Equation (1) can be written as
→ F q
or a = = [B0v y $i + (E 0 – v x B0) $j ] dv x E 
m m = αB0  0  sin ωt
→ dt  B0 
or a = α [B0 v y $i + (E 0 – v x B0) $j] vx t

dv x
or ∫0 dv x = αΕ 0 ∫ 0 sin ωt dt
i.e., ax = = αB0 v y …(1) αE 0 E
dt or v x = (1 – cos ωt) = 0 (1 – cos ωt)
dv y ω B0
and ay = = α (E 0 – v x B0) …(2)
dt dx E 0
or = (1 – cos ωt)
Differentiating equation (2) with respect to dt B0
time x E t
d 2v y dv
or ∫ 0 dx = B00 ∫ 0 (1 – cos ωt) dt
= – αB0 x = – α 2B02v y (from 1)
dt 2 dt E0 E0
d 2v y or x= t– sin ωt
or = – ω 2v y where ω = αB0 B0 B0 ω
dt 2 E0
= [ωt – sin ωt]
i.e., v y oscillates simple harmonically with B0 ω
angular frequency ω. Hence v y can be written
E0
as = (ωt – sin ωt) ω = αB0
v y = v 0 sin ωt, because v y = 0 at t = 0 αB02
dv y E0
∴ = v 0 ω cos ωt …(3) or x = a (ωt – sin ωt) a=
dt αB02
dv y therefore, x = a (ωt – sin ωt)
At time t = 0, = v 0 ω from equation (3)
dt
dv y and y = a (1 – cos ωt)
and = αE 0 from equation (2) because
dt E0
where ω = αB0 and a=
v x = 0 at t = 0. αB02
∴ v 0 ω = αE 0 Magnetic force due to B0 cos θ component on
α E0 E0 the whole loop will be zero.
or v0 = = (ω = α B0)
ω B0 6. Magnetic force on a small current carrying
E0 element dl would be
Therefore, v y = sin ωt …(4) Y
B0
dy E 0
or = sin ωt B0 sinθ X
dt B0 θ
y E t
or ∫ 0 dy = B00 ∫ 0 sin ωt dt θ
dl 2 +d
2

a a
E0 (0,0,d)
or y= (1 – cos ωt) B0 cosθ θ
B0 ω a d
E0 Z
= 2
(1 – cos ωt) i
αB0
E0 dFm = i (B0 sin θ) dl
y = a (1 – cos ωt) where a =
αB0
2
∴ Total magnetic force will be
364
Fm = iB0 sin θ Σdl 8. (a) Magnetic force on dl will be
a 2πa 2iB0 dFm = iB ( dl)
= i B0 (2πa) =
a2 + d 2 a2 + d2 From Fleming’s left hand rule, × ×dt →
×
2πr → × × ×
7. One side of the polygon is a = direction of d Fm will be inwards × × → ×
n dFm
O (towards the centre). × × ×
(b) F is the force of compression in the wire.
For equilibrium of element
α β
F F

r dθ/2 i dθ/2
i dFm

Q P dl = adθ
a/2 dθ

1  2π  π
α =β =  =  dθ 
2 n n 2F sin   = dFm
 2
a  πr  π
r= cot α =   cot  dθ   dθ dθ 
2 n n or 2F   = iB(adθ)  sin ≈ 
 2  2 2
Magnetic field due to one side PQ at centre is
µ i or F ⋅ dθ = iB (a dθ)
B = 0 (sin α + sin β)
4π r or F = iBa
µ i  π (c) When the magnetic field is switched off,
B= 0  2 sin 
4 π  πr   π  n only unbalanced force is the compression
  cot  
n n force F. Due to which the length of the
µ 0 in sin (π/n) tan (π/n) wire will increase and hence, radius will
= increase, length of wire is
2π 2r
∴ Net magnetic field due to all n sides will be l = 2πa
Bnet = nB ∴ dl = (2π) ⋅ da
µ i n2 sin ( π/n) tan ( π/n) or increase in radius da =
dl
Bnet = 0 …(1)
2π 2 r 2π
(b) Expression of Bnet can also be written as F ⋅l
where dl = increase in length =
µ 0 i  sin (π/n) 1
2 (πr 2) ⋅ Y
Bnet =   ⋅
2r  (π/n)  (cos π/n)  Fl 
Y = 
now as n → ∞, π/n → 0,  A ∆l 
sin π/n (iBa) (2πa) 2iBa 2
cos π/n → 1 and →1 = =
π/n (πr ) Y2
Yr 2
µ i
∴ Bnet = 0 Substituting this in equation (1), increase in
2r
Which is the magnetic field at the centre of a radius
circular coil, because as n → ∞, polygon ia 2 B
da =
becomes a circle of radius r. πr 2 Y
9. First consider the force on 3 due to 1. B cos θ The magnitude of this force is :
has no effect on the force onwire 3. F = idB = (48) (012
. ) (0.24) N
i1
+ or F = 1.3824 N
Now as there is no slipping, frictional force (f)
θ
r
will act in backward
l1 direction and instan- F
taneous axis of rotation
B cosθ x=l will be point of contact. f
Angular acceleration α
x=0 θ i3
about which is eual to :
B sinθ
B τ F. R
dx α= =
x Ι 1 2
 MR + MR 
2
l 2 
∴ F1 = ∫ (i 3) (B sin θ) dx 2F
0 = (R = Radius)
l
3MR
µ i   x 
= ∫ (i 3)  0 1    ⋅ dx and linear acceleration of COM of rod is :
 2π r   r 
0 2F
l a = Rα =
µ 0 i1i 3 x 3M
=
2π ∫ x 2 + l 2 ⋅ dx 2 (1.3824)
a
0 1 =
with proper substitutions we can find that 3 (072
. ) α
 2 = 1.28 m/s2
µ ii l
F1 = 0 1 3 ln  1 +    (inwards)
2π   l1   ∴ speed of rod as it leaves the rails is :
 
v = 2aL
similarly
= 2 × 1.28 × 0.45 = 1.07 m/s
µ 0 i2 i3  2
l
F2 = ln  1 +    (outwards) 11. Magnetic field is in negative z-direction i.e.,
2π  l2  perpendicular to paper inwards. Magnetic
 

Therefore, net force will be force F mcan not change the speed of a
F = F1 – F2 particle, therefore, the Y
 2 speed of the particle v0
µ 0 i3  l remains the same
= i1 ln 1 +   X
2π   l1  throughout. Let the
 particle makes an angleθ
2 with the positive Z
 l
– i 2 ln 1 +    (inwards) x-direction at x = x . Then
 l2  
 dx
vx = = v 0 cosθ …(i)
10. Magnetic force on the rod can be assumed to dt
be acting at the centre of the rod, rightwards. F (B x ) qv 0
Now am = m = 0 = B 0 v 0 xα
m m
a y = a m cosθ
 dv y 
F or   = (B0 v 0 xα) cos θ
f  dt 

L
Y
+ + + + Z
Y-direction is
Fm perpendicular
θ v0
θ θ r to paper inwards
+ + + + v0
θ
+ a + + +
v0 Fm y
X v0
X
+ x + + +
 dv y   dx  x
or     = (B0 xα) (v 0 cos θ) 2π m 2π
 dx   dt  T = =
B0q B0α
dv y  dx 
or = (B0 xα)  = v 0 cosθ 2π
dx  dt  or angular velocity ω = = B0 α
T
or dv y = B0 x α ⋅ dx
v0 x max
Angle rotated by particle in time t is
θ = ωt = B0αt
or ∫ dvy = B0α ∫ x dx …(3)
0 0 v x = v 0 cos θ = v 0 cos B0 αt …(4)
B α 2 v z = v 0 sin θ = v 0 sin B0 αt …(5)
or v 0 = 0 x max
2 mv 0
x = r sin θ = sin B0 αt
2v0 B0 q
or x max =
B0α v0
= sin B0 αt …(6)
12. (a) Electric field is in y-direction, therefore, B0 α
→ and z = r (1 – cos θ)
electrostatic force Fe (which is also in
v
y-direction) will provide an acceleration to = 0 (1 – cos B0 αt) …(7)
the particle in y-direction. Therefore, B0 α
y-component of its velocity will go on From equations (1), (4) and (5) velocity of
increasing. Magnetic field is also in particle at time t is
→ →
y-direction so, magnetic force F mwill act v = v x i$ + v y $j + v zk$
in x – z plane or the particle will rotate in
a circle with its plane in x – z plane. →
or v p = v 0 cos ( B0 αt) i$ + ( E 0 αt) $j
Hence the path of the particle will
be a helix with increasing pitch. + v 0 sin ( B0 αt) k$
Fy qE 0 q 
(b) a y = = = E 0α  = α (c) From equations (2), (6) and (7) position
m m m  of particle at time t is
∴ v y = a y t = E 0 αt …(1) →
r = xi$ + yj$ + zk$
p
1 1
and y = a y t 2 = E 0αt 2 …(2)
2 2 → v0 1
or rp = sin ( B0 αt) i$ + E 0 αt 2 $j
At time t = 0, particle is at origin. Its velocity is B0 α 2
in x-direction so magnetic force will be in v0
positive z-direction. The particle will rotate in + (1 – cos B0 αt) k$
B0 α
x – z plane with time period
(d) Particle will touch the y-axis for n th time P1
2nπ
after time t = nT = .
B0α
r
So, y-co-ordinate at that moment will be
1
y n = E 0 αt 2
2
2
1  2nπ 
= E0 α  
2  B0α 
→  µ   2I 
2n 2 π 2 E 0 Here B1 =  0    towards left
or yn =  2π   r 
B02 α
→  µ   I/ 2 
(e) y-compoment of velocity at that moment B2 =  0    towards right
 2π   r – a/ 2
will be
 2nπ  2nπE 0 → µ   I/ 2 
v y = E 0 αt = (E 0 α)   = and B3 =  0    towards right
 B0 α  B0  2π   r + a/ 2

v x2 + v z2 = v 0 i.e., in x – z plane speed v 0 Hence net magnetic field at P1 is


µ I 1 1 1 
remains constant. B= 0  – –
π  r 4 (r – a/ 2) 4 (r + a/ 2)
Y vy v
towards left
µ I 1 1 1 
φ = 0  – –
v0
π  r (4r – 2a) (4r + 2a)
X – Z plane
towards left
v0 µ I  2r 2 – a 2 
tan φ = B= 0  2  to the left
vy π r  4r – a 2 
v0  v B  (b) In the similar manner magnetic field at P2
or φ = tan −1 = tan −1  0 0 
vy  2nπE 0  will be
B = B1 – 2B2 cosθ
13. (a) Equal and opposite currents can be B1
assumed in the cavities.
r2+a2
a 4
+ 2 + 3 2 r
1 θ
θ

I current I current
2I current
2 inwards 2 inwards
outwards θθ
B2 B3
The current in cavity will be
 I  µ0
 
2    a 
2
I  2I  µ0
i= 2 =   – 2 
 a    π    = 2π r  2π 
 πa – 2 (π)  2   2  2
 
 I/ 2   r 
Total magnetic field at P1 is    
×  2 a2  2
 2 a 
→ → → →  r +   r + 
B = B1 + B2 + B3  4  4
µ0 I 1 2r  3T0 ω BQR 2  mg 
=  – 2  = T0 + max  = T 0
π r + a2  2 2D  2 
 4 r
DT0
µ 0 I ( 2r 2 + a 2 ) ∴ ωmax =
= BQR 2
π r ( 4r 2 + a 2 )
16. (a) and (c) Let the direction of current in wire
towards the top of the page.
PQ is from P to Q and its magnitude be I.
2qVm z
14. r=
Bq
m
or r∝
q
P Q y
rP mP qα
∴ = I
rα mα qP
S
x R
1 2 1
= = The magnetic moment of the given loop is :
4 1 2

15. In equilibrium, M = − Iabk$
2T0 = mg Torque on the loop due to magnetic force is :
mg → → →
or T0 = …(1) τ1 =M×B
2
Magnetic moment, = (− Iabk$) × {(3i$ + 4k$) B0}
ω  = − 3IabB0 $j
M = iA =  Q (πR 2)
 2π  Torque of weight of the loop about axis PQ is :

ωBQR 2 → →
τ = MB sin 90° = τ2 =r ×F
2
a  mga $
Let T1 and T2 be the tensions in the two strings =  i$ × (−mgk$) = j
2  2
when magnetic field is switched on. (T1 > T2)
For translational equilibrium of ring in vertical We see that when the current in the wire PQ is
direction, → →
from P to Q, τ1 and τ 2 are in opposite direction.
T1 + T2 = mg …(2) So they can cancel each other and the loop
For rotational equilibrium, may remain in equilibrium. So the direction of
current I in wire PQ is from P to Q. Further for
D ωBQR 2
(T1 − T2) =τ= equilibrium of the loop :
2 2
→ →
ωBQR 2 | τ1| = | τ 2|
or T1 − T2 = …(3)
D mga
or 3IabB0 =
Solving Eqs. (2) and (3) we have 2
mg
mg ωBQR 2 I =
T1 = + 6 bB0
2 2D
(b) Magnetic force on wire RS is :
As T1 > T2 and maximum values of T1 can be
→ → →
3T0
, we have F = I ( l × B)
2
= I [(− bj$) × {(3i$ + 4k$) B0}] Force on CD :
→ Current in central wire is also i = 10 A
or F = IbB0 (3 k$ − 4i$) Magnetic field at P due to central wire,
17. (a) Given i = 10 A, r 1= 0.08 m and µ i
B= 0.
r 2= .012 m. Straight portions i. e., CD etc. 2π x
will produce zero magnetic field at the ∴ Magnetic force on element dx due to this
centre. Rest eight arcs will produce the
magnetic field
magnetic field at the centre in the same
µ i
direction i. e., perpendicular to the paper dF = (i)  0 .  . dx (F = ilB sin 90° )
outwards or vertically upwards and its  2π x 
magnitude is  µ  dx
=  0  i2
D  2π  x
C Therefore, net force on CD is
x = r2
µ 0i2 0.12
dx µ 0 2  3
r1 A
F= ∫ dF = 2π ∫ x
=

i ln  
 2
x = r1 0. 08
r2 D
i
B
i
P
B = Binner arcs + Bouter arcs dx
C
1 µ i 1 µ 0 i
=  0 +  
2  2r1  2  2r2 
x
µ  r + r 
=  0  (πi)  1 2 
 4π  r1r2 
Substituting the values, we have Substituting the values,
. ) (10) (0.08 + 012
(10–7 ) (314 . ) F = (2 × 10– 7 ) (10)2 ln (1.5)
B= tesla
(0.08 × 012
. ) or F = 8.1 × 10 – 6 N (inwards)
B = 6.54 × 10 –5
tesla Force on wire at the centre : Net
(Vertically upward or outward normal to the magnetic field at the centre due to the circuit is
paper) in vertical direction and current in the wire in
(b) Force on AC : Force on circular centre is also in vertical direction. Therefore,
portions of the circuit i. e. , AC etc. due to net force on the wire at the centre will be zero.
the wire at the centre will be zero because (θ = 180° ). Hence
magnetic field due to the central wire at (i) Force acting on the wire at the centre is
these arcs will be tangential (θ = 180° ) as zero.
shown. (ii) Force on arc AC = 0.
(iii) Force on segment CD is 81 . × 10– 6 N
(inwards).

18. (a) Magnetic field ( B ) at the origin
= Magnetic field due to semicircle KLM
+ Magnetic field due to other
semicircle KNM
370
→ µ I µ I 19. (a) θ = 30°,
∴ B = 0 (– i$) + 0 ($j )
4R 4R L
sin θ =
→ µ I µ I µ0 I $ $ R
B = 0 i$ + 0 $j = (– i + j ) mv 0
4R 4R 4R Here R=
B 0q
∴ Magnetic force acting on the particle
→ → → L
∴ sin 30° =
F = q( v × B) mv 0
µ I B 0q
= q {(– v 0 i$) × (– i$ + $j )} 0
4R 1 B0qL
or =
→ µ qv I 2 mv 0
F = – 0 0 k$
4R mv 0
∴ L=
→ → → 2B 0 q
(b) F KLM = F KNM = F KM
x=0 x=L
X X
→ + + + + Z
and F KM = BI (2R) i$ = 2BIR i$ B=B0k
C
→ → + + + Y
∴ F1 = F2 = 2BIRi$ θ R v0
+ + + +
or Total force on the Loop,
P
→ → → → + +
L
+ +
F = F1 + F2 or F = 4BIRi$
A + + + +
If a current carrying wire ADC (of any shape) q v0

+ + + +
is placed in a uniform magnetic field B . Y
perpendicular to the plane ADC. R
→ → (b) In part (i)
+

+
Then F ADC = F AC L v0
sin 30° =
→ R
B
+

+
or | F ADC| = I ( AC)B
1 L
or =
From this we can conclude that net force on a 2 R
+

+
A v0
current carrying loop in uniform magnetic R
field is zero. In the question, segments KLM or L=
+

+
2
and KNM also form a loop and they are also L′ = 2 R > R
2.1
21
.
placed in a uniform magnetic field but in this Now when L′ = 21
. L or R
case net force on the loop will not be zero. It 2
would had been zero if, the current in any of ⇒ L′ > R
the segments was in opposite direction.
Therefore, deviation of the particle is θ = 180°
is as shown.
+

B
→ →
D i C ∴ v f = – v 0 i$ = v B
+

T πm
and t AB = =
+

A 2 B0 q
+

+
Solutions 371
→ → → → → →  → 
E B $ v0 v × B →  qB   v 0  qE  E
501. j =
$ or :i = or k = 0
$ ∴ v = v 0 cos  t   + t 
E B v0 v0 B m   v0  m  E
Force due to electric field will be along Y-axis.    
Magnetic force will not affect the motion of → → 
charged particle in the direction of electric field  qB  v × B 
+ v 0 sin  t  0
(or Y-axis). So, m   v 0B 
Y  
E and B
→  qB  → q  →
or v = cos  t (v 0 ) +  t ( E )
 m  m 
X → →
v0  qB   v 0 × B 
+ sin  t
 m   B 
 
Z
Fe qE The path of the particle will be a helix of
ay = = = constant. increasing pitch. The axis of the helix will be
m m
along Y-axis.
qE
Therefore, v y = a y t = .t …(1) 21. Magnetic moment of the loop,
m →
The charged particle under the action of M = (iA) k$ = (I 0L2) k$
magnetic field describes a circle in x-z plane Magnetic Field,

(perpendicular to B ) with →
Z B = (B cos 45° ) i$ + (B sin 45° ) $j
vz v0 B $ $
= (i + j )
θ 2
vx
(a) Torque acting on the loop,
→ → → B $ $j )
X
v0
τ = M × B = (I 0L2k$) ×  (i + 
 2
2πm 2π qB
T = or ω = = → I L2 B
Bq T m ∴ τ = 0 ( $j – i$)
2
Initially (t = 0) velocity was along X-axis.
→ →
Therefore, magnetic force (Fm ) will be along or | τ | = I 0L2B
→ → → (b) Axis of rotation coincides with the torque
positive Z-axis [Fm = q (v 0 × B )]. Let it makes
and since torque is in $j – i$ direction or
an angle θ with X-axis at time t, then θ = ωt Y
 qB 
∴ v x = v 0 cos ωt = v 0 cos  t …(2)
m  S R

 qB 
v z = v 0 sin ωt = v 0 sin  t …(3)
m  X

From (1), (2) and (3)


P Q

∴ v = v x i$ + v y $j + v z k$
parallel to QS. Therefore, the loop will
rotate about an axis passing through Q
i i i
and S as shown alongside :
x =x
→ O x
| τ|
Angular acceleration, α = d–x
I d+x
Where I = moment of inertia of loop about
QS. 1 2 3

I QS + I PR = I ZZ B=0 V B=0
(From theorem of perpendicular axis)
But I QS = I PR
4
∴ 2I QS = I ZZ = ML2
3
2
∴ I QS = ML2
3 d
x=– d B=0 x=–
→ 3 3
| τ | I 0L2B 3 I 0B z=0 z=0
∴ α= = =
I 2 Let magnetic field is zero on line z = 0 and
ML2 2 M
3 x = x . Then magnetic field on this line due to
∴ Angle by which the frame rotates in time ∆t wires 1 and 2 will be along negative z-axis and
is due to wire 3 along positive z-axis. Thus
1 B1 + B2 = B3
θ = α . (∆t)2 µ i µ i µ i
2 or 0 + 0 = 0
3 I 0B 2π (d + x ) 2π x 2π (d − x )
or θ= . ( ∆t) 2 1 1 1
4 M or + =
22. (i) Magnetic field will be zero on the d+x x d–x
y-axis i. e., x = 0 = z This equation gives
d
I II
Y
III IV
x=±
3
i i i Hence there will be two lines
d d
X x = and x = – (z = 0)
x = –d O x = +d 3 3
where magnetic field is zero.
(ii) In this part we change our coordinate axes
system, just for better understanding.
Z x y-axis
Magentic field can not be zero in region I and
region IV because in region I magnetic field
will be along positive z direction due to all the
three wires, while in region IV magnetic field
1 2 3
will be along negative z-axis ⊗ due to all the
x x x X
three wires. It can be zero only in region II and
III. x = –d x=0 x=d
There are three wires 1, 2 and 3 as shown in 1 a 1 i µ0
figure. If we displace the wire 2 towards the = =
2π z 2π d πλ
z-axis, then force of attraction per unit length
between wires (1 and 2) and (2 and 3) will be i µ0
or f =
given by 2π d πλ
µ 0 i2 23. Given that x >> a.
F=
2π r Z
k
Y
The components of F along x-axis will be j
cancelled out. Net resultant force will be
v i
towards negative z-axis (or mean position) X
and will be given by a
x
2
x
F θ
F Magnetic field at the centre of the coil due to
θ
the bar magnet is,
r r
z µ 2M µ M
B= 0 3 = 0 3
1 3 4π x 2π x
x x
d d Due to this, magnetic flux linked with the coil
will be,
µ i2 z
Fnet = 2F cos θ = 2  0  µ0 M µ 0Ma 2
φ = B. S = (π a 2
) =
 2π r  r 2π x 3 2x 3
µ0 i2 ∴ Induced emf in the coil, due to motion of
Fnet = .z
π (z 2 + d 2) the magnet is
dφ  µ Ma 2  d  1 
If z << d, then e=– =– 0   
µ i2  dt  2  dt  x 3 
z 2 + d2 ≈ d2 and Fnet = –  0 2  . z
 π d  µ 0Ma 2  3  dx 3 µ 0 Ma
2
 dx 
=  4 . = v  = v
2  x  dt 2 x 4  dt 
Negative sign implies that Fnet is restoring in
nature. Therefore, induced current in the coil is,
Therefore, Fnet ∝ – z e 3 µ 0 Ma 2
i= = v
i. e., the wire will oscillate simple harmonically. R 2 Rx 4
Let a be the acceleration of wire in this Magnetic moment of the coil due to this
position and λ is the mass per unit length of induced current will be,
wire then
3 µ 0 Ma 2
µ i2  M ′ = iS = v (πa 2)
Fnet = λ . a = –  0 2 z 2 Rx 4
 π d 
3 µ 0π Ma 4v
M′ =
µ i  2
2 Rx 4
or a = –  0 2 . z
 πλd  → →
Potential energy of M ′ in B will be
∴ Frequency of oscillation
U = – M ′ B cos 180°
1 Acceleration
f = 3 µ 0π Ma 4v  µ 0 M 
2π Displacement U = M′ B =  . 
2 Rx 4  2π x 3 
i → → →
B
(ii) τ = M × B
M
or τ = MB sin θ θ
v
where θ is the angle
S N B
due to → →
M′ between M and B
magnet
(of coil)
θ = 30°
θ=30°
 eh 
∴τ =   (B) sin 30°
2  4 πm 
3 µ 0 M 2a 4v 1
U=
4 R x7 ehB
∴τ =
2 8 πm
dU 21 µ 0 M 2 a 4v 2
∴ F= = → →
dx 4 Rx 8 The direction of τ is perpendicular to both M
Positive sign of F implies that there will be a →
repulsion between the magnet and the coil. and B .
25. Let m be the mass per unit length of wire AB.
Note that here we can not apply
At a height x above the wire CD, magnetic
µ 6MM ′
F= 0 (directly) …(1) force per unit length on wire AB will be given by
4π x 4 Fm
because here M ʹ is a function of x. However
A B i1 = 20 A
equation (1) can be applied where M and M ʹ
both are constants. x=d
Fg
= 0.01 m
24. (i) In ground state (n = 1), according to
i2 = 30 A
Bohr's theory : C D

mvR =
h
or v =
h µ 0 i1i 2
Fm = (upwards) …(1)
2π 2πmR 2π x
Now time period, Weight per unit length of wire AB is
2π R 2π R 4 π 2mR 2 Fg = mg (downwards)
t= = =
v h/ 2πmR h Here m = mass per unit length of wire AB
Magnetic moment M = i A At x = d, wire is in equilibrium i. e. ,
Charge µ 0 i1i 2
where i = Fm = Fg or . = mg
Time period 2π d
µ 0 i1i 2 mg
e eh or = …(2)
= = 2π d 2 d
4 πmR 2 4 π 2mR 2
h When AB is depressed, x decreases therefore,
Fm will increase, while F remains the same.
and A = πR 2
Let AB is displaced by dx downwards.
 eh  Differentiating (1) w.r.t. x, we get
∴ M = (πR 2)  2 2 
 4 π mR  µ ii
dFm = – 0 . 1 22 . dx …(3)
eh 2π x
or M =
4πm i. e., restoring force,
→ F = dFm α – dx
Direction of magnetic moment M is
Hence the motion of wire is simple harmonic.
perpendicular to the plane of orbit.
From equations (2) and (3), we can write
y
 mg 
dFm = –   . dx (x = d) I +
B
is the positive Z-direction and
 d  is the negative Z-direction
g a dx
∴ Acceleration of wire a = –   dx
 d θ P i
O x x
Hence period of oscillation C
θθ
a
dx displacement BA
T = 2π = 2π BB
a acceleration I +A Bnet

d 0.01 y′ L
or T = 2π = 2π
g 9.8 → → → µ i
or T = 0 .2 s | B A| = | BB| = | B| = 0
2π AP
26. Kinetic energy of electron, S Therefore, net magnetic field at P will be along
1 negative y-axis as shown
K = mv 2= 2 keV B
2 → µ  i  x 
∴ speed of electron, and Bnet = 2| B| cos θ = 2  0   
G
60°
v
 2π  AP  AP 
2K
v=  µ  i. x µ ix
m Bnet =  0  = 0. 2
 π  ( AP)2
π (a + x 2)
2 × 2 × 1.6 × 10−16
v= m/s Therefore, force on the element will be
. × 10−31
91
(F = ilB)
v = 2.65 × 107 m/s µ ix 
→ dF = i  0 2  dx (in negative z-direction)
 π a + x 2
Since the velocity (v) of the electron makes an
→ ∴ Total force on the wire will be
angle of θ = 60° with the magnetic field B , the x =L L
µ 0 i2 xdx
path will be a helix. So, the particle will hit S if F= ∫ dF =
π ∫ x 2 + a2
GS = np Here n = 1, 2, 3, K x =0 0

2πm µ i  L2 + a 2 
2
p = pitch of helix = v cos θ = 0 ln   (in negative z-axis)
qB 2π  a2 
But for B to be minimum, n = 1 →
2πm µ i2  L2 + a 2  $
Hence GS = p = v cos θ Hence F = – 0 ln   k
qB 2π  a2 
2πmv cos θ (b) If current in wire B is reversed, then
∴ B = B min =
q(GS) magnetic fields due to A and B will be in the
Substituting the values, we have directions shown in figure. i. e., net magnetic

 1 field Bnet will be along positive x-axis and
(2π)(91
. × 10– 31) (2.65 × 107 )  
 2
B min = tesla since current is also along positive x-axis,
(1.6 × 10– 19) (01
.) force on wire OC will be zero.
Y
or B min = 4.73 × 10 – 3 tesla
B
27. (a) Let us assume a segment of length dx at a BB
point P, a distance x from the centre i P C
shown in figure. X
Bnet
Magnetic field at P due to current in wires A
and B will be in the directions perpendicular to A + BA
AP and BP respectively as shown. →
Note: B A is not necessarily parallel to BP.

You might also like